Find the exact values of x and y.​

Find The Exact Values Of X And Y.

Answers

Answer 1

The values of x and y are given as follows:

x = y = 5.

What is the Pythagorean Theorem?

The Pythagorean Theorem states that in a right-angled triangle, the square of the length of the hypotenuse (the longest side) is equal to the sum of the squares of the lengths of the other two sides.

The theorem is expressed as follows:

c² = a² + b².

In which:

c is the length of the hypotenuse.a and b are the lengths of the other two sides (the legs) of the right-angled triangle.

The diagonal length of the rectangle is the hypotenuse of a right triangle of sides 6 and 8, hence:

d² = 6² + 8²

d² = 100

d = 10.

The segments x and y are each half the length of the diagonal, and the two diagonals have the same length for a rectangle, hence:

x = y = 5.

More can be learned about the Pythagorean Theorem at brainly.com/question/30203256

#SPJ1


Related Questions

Find the first-order and the second-order Taylor formula for f(x, y) = 17e(x+y) at (0,0). (Use symbolic notation and fractions where needed. ) f(x, y) = f(x, y) =

Answers

The first-order and the second-order Taylor formula for f(x, y) = 17e(x+y) at (0,0) is f(x,y) = 17 + 17x + 17y + (17/2)x² + 17xy + (17/2)y²

The first-order Taylor formula for f(x,y) = 17[tex]e^{(x+y)}[/tex] at (0,0) is:

f(x,y) ≈ f(0,0) + ∇f(0,0) · (x,y)

≈ 17[tex]e^{(0+0)}[/tex] + (∂f/∂x, ∂f/∂y)(0,0) · (x,y)

≈ 17 + (17,17) · (x,y)

≈ 17 + 17x + 17y

The second-order Taylor formula for f(x,y) = 17[tex]e^{(x+y)}[/tex] at (0,0) is:

f(x,y) ≈ f(0,0) + ∇f(0,0) · (x,y) + (1/2)(x,y) · Hf(0,0) · (x,y)

≈ 17 + (17,17) · (x,y) + (1/2)(x,y) · ( ∂²f/∂x² ∂²f/∂x∂y ; ∂²f/∂y∂x ∂²f/∂y² ) (0,0) · (x,y)

≈ 17 + 17x + 17y + (1/2)(x,y) · (17 17 ; 17 17) · (x,y)

≈ 17 + 17x + 17y + (1/2)(17x² + 34xy + 17y²)

≈ 17 + 17x + 17y + (17/2)x² + 17xy + (17/2)y²

Learn more about the second-order Taylor formula at

https://brainly.com/question/31401542

#SPJ4

a bowl contains three red and four yellow marbles. you randomly select two marbles from the bowl. which of the following is a conditional probability? assume the second marble is drawn from the marbles remaining after the first draw.

Answers

The conditional probability in this scenario is the probability of drawing a yellow marble on the second draw, given that the first marble drawn was red.

To calculate this conditional probability, we can use Bayes' theorem, which states that the probability of an event (in this case, drawing a yellow marble on the second draw) given some prior knowledge (in this case, that the first marble drawn was red) is equal to the probability of both events occurring (drawing a red marble first and a yellow marble second) divided by the probability of the prior event (drawing a red marble first).

The probability of drawing a red marble first is 3/7 since there are three red marbles out of a total of seven marbles in the bowl. Once a red marble is drawn, there are six marbles remaining, of which three are yellow. Therefore, the probability of drawing a yellow marble second, given that the first marble was red, is 3/6 or 1/2.

Putting this together, we can calculate the conditional probability as follows:

P(Yellow on Second Draw | Red on First Draw) = P(Red on First Draw and Yellow on Second Draw) / P(Red on First Draw)
= (3/7) * (3/6) / (3/7)
= 1/2

Therefore, the conditional probability in this scenario is 1/2 or 50%. This means that there is a 50% chance of drawing a yellow marble on the second draw, given that the first marble drawn was red.

To learn more about Probability, visit:

https://brainly.com/question/27342429

#SPJ11

Now answer the question:
Claire and her children went into a grocery store and she bought $8 worth of apples
and bananas. Each apple costs $1 and each banana costs $0.50. She bought a total of
11 apples and bananas altogether. Determine the number of apples, x, and the
number of bananas, y, that Claire bought.

Answers

So if she bought a total of $8 worth that means there is more than one possibility but it says apples and bananas total but I’m gonna do more than that

For a total of $8 she could by 16 bananas and 0 apples

For $8 she could by 8 apples and zero bananas

For $8 she could by 4 apples and 8 bananas

Show that the functions f(x) = x, g(x) = x - 1, and h(x) = x + 3 are linearly dependent. Show, however, that f(x) = x2, g(x) = x - 1, and h(x) = x + 3 are linearly independent

Answers

To show that the functions f(x) = x, g(x) = x - 1, and h(x) = x + 3 are linearly dependent, we need to find a non-zero linear combination of the three functions that equals zero.

Let's assume that a, b, and c are constants such that:

a*f(x) + b*g(x) + c*h(x) = 0

Substituting in the given functions, we get:

a*x + b*(x - 1) + c*(x + 3) = 0

Simplifying this equation, we get:

(a + b + c) * x + (-b + 3c) = 0

For this equation to hold true for all x, we must have:

a + b + c = 0

-b + 3c = 0

This is a system of two equations with three unknowns, which means that we have infinitely many solutions. For example, we could choose a = 1, b = -2, and c = 1, and the equation would hold true. Therefore, we have shown that the functions f(x) = x, g(x) = x - 1, and h(x) = x + 3 are linearly dependent.

Now, let's show that the functions f(x) = x^2, g(x) = x - 1, and h(x) = x + 3 are linearly independent.

We need to show that there are no non-zero constants a, b, and c such that:

a*f(x) + b*g(x) + c*h(x) = 0

Substituting in the given functions, we get:

a*x^2 + b*(x - 1) + c*(x + 3) = 0

This equation holds true for all x if and only if its coefficients are all zero. Therefore, we need to solve the system of three equations:

a = 0

-b + c = 0

3c = 0

The first equation tells us that a must be zero. The third equation tells us that c must be zero. Substituting c = 0 into the second equation, we get:

-b = 0

Therefore, we must have b = 0 as well.

Since a, b, and c are all zero, we have shown that the functions f(x) = x^2, g(x) = x - 1, and h(x) = x + 3 are linearly independent.

Learn more about functions: https://brainly.com/question/11624077

#SPJ11

suppose there is a lottery where the organizers pick a set of 11 distinct numbers. a player then picks 7 distinct numbers and wins when all 7 are in the set chosen by the organizers. numbers chosen by both the players and organizers come from the set {1, 2, ..., 80}. (a) let the sample space, s, be all the sets of 7 numbers the player can choose. what is |s|? (b) let e be the event that all the numbers the player chooses are in the winning set. what is |e|? (c) what is the probability of winning? as a reminder, you may leave your answer un- simplified.

Answers

(a) 40,475,358.

(b) 330
(c) 0.0008%.

(a) To find |S|, the total number of sets of 7 distinct numbers a player can choose, we need to find the combinations of choosing 7 numbers from the 80 available options. This can be calculated using the combination formula:

C(n, k) = n! / (k! * (n - k)!)

In this case, n = 80 (total numbers) and k = 7 (numbers to choose). So, |S| = C(80, 7):

|S| = 80! / (7! * (80 - 7)!)
|S| = 80! / (7! * 73!)

(b) To find |E|, the number of sets where all 7 numbers chosen by the player are in the winning set of 11 numbers chosen by the organizers, we need to find the combinations of choosing 7 numbers from the 11 available options in the winning set:

|E| = C(11, 7)
|E| = 11! / (7! * (11 - 7)!)
|E| = 11! / (7! * 4!)

(c) To find the probability of winning, we need to calculate the ratio of the favorable outcomes (|E|) to the total possible outcomes (|S|):

P(winning) = |E| / |S|

P(winning) = (11! / (7! * 4!)) / (80! / (7! * 73!))

Learn more about :

Distinct numbers : brainly.com/question/24049358

#SPJ11

No current will flow between two charged bodies if they have the
same
A) resistance
B) charge
C) potential
D) charge/ potential ratio

Answers

Two bodies can have the same resistance or charge/potential ratio, but still have different potentials, resulting in the flow of current between them.

The correct answer is C) potential.

When two bodies have the same potential, it means that the electric potential difference between them is zero. In this case, no work is required to move a charge from one body to the other, because the potential energy of the charge is the same on both bodies.

Since current is defined as the flow of electric charge, if there is no potential difference between two bodies, there will be no force driving the charges to move from one body to the other. Hence, no current will flow between the two bodies.

It is important to note that having the same resistance or charge/ potential ratio does not necessarily mean that no current will flow between two bodies. Resistance refers to the opposition to the flow of current, and the charge/ potential ratio is the charge per unit of electric potential. Therefore, two bodies can have the same resistance or charge/potential ratio, but still have different potentials, resulting in the flow of current between them.

To learn more about resistance visit

https://brainly.com/question/15301680

#SPJ11

Brewsky's is a chain of micro-breweries. Managers are interested in the costs of the stores and believe that the costs can be explained in large part by the number of customers patron¬izing the stores. Monthly data regarding customer visits and costs for the preceding year for one of the stores have been entered into the regression analysis and the analysis is as follows:Average monthly customer-visits 1,462Average monthly total costs $ 4,629Regression Results Intercept $ 1,496b coefficient $ 2.08R2 0.868141. In a regression equation expressed as y = a + bx, how is the letter b best described? (CMA adapted)a. The proximity of the data points to the regression line.b. The estimate of the cost for an additional customer visit.c.The fixed costs per customer-visit.d.An estimate of the probability of return customers.2. How is the letter x in the regression equation best described? (CMA adapted)a. The observed customer visits for a given month.b. Fixed costs per each customer-visit.c. The observed store costs for a given month.d. The estimate of the number of new customer visits for the month3. What is the percent of the total variance that can be explained by the regression equation? (CMA adapted)a. 86.8%b. 71.9%c. 31.6%d. 97.7%

Answers

In this regression analysis, the letter b in the equation y = a + bx represents the estimate of the cost for an additional customer visit. This means that for every additional customer visit to the store, the expected increase in monthly total costs is $2.08, according to the regression model.

The letter x in the regression equation represents the observed customer visits for a given month. This means that the regression model is predicting the monthly total costs based on the number of customer visits in that month.

The R2 value of 0.8681 means that 86.81% of the total variance in the monthly total costs can be explained by the regression equation, which indicates a strong relationship between the number of customer visits and the total costs. This can help managers of Brewsky's make informed decisions about how to allocate resources and improve profitability. However, it is important to note that other factors may also influence the costs, and the regression model may not capture all of these factors.

To know more about regression analysis refer here:

https://brainly.com/question/30011167

#SPJ11

in a single statement: declare, create and initialize an array named a of 10 elements of type int with the values of the elements (starting with the first) set to 10 , 20 , ..., 100 respectively.

Answers

If you provide more values than the size of the array, you'll get a compilation error.

In C or C++ programming languages, an array can be declared, created, and initialized in a single statement. Here's how you can declare, create, and initialize an array named a of 10 elements of type int with the values of the elements (starting with the first) set to 10, 20, 30, 40, 50, 60, 70, 80, 90, and 100, respectively:

int a[10] = {10, 20, 30, 40, 50, 60, 70, 80, 90, 100};

This statement does the following:

Declares an array named a of 10 elements of type int.

Initializes the elements of the array with the specified values in the curly braces, starting from the first element.

Note that if you don't provide enough values in the curly braces, the remaining elements will be initialized to 0. If you provide more values than the size of the array, you'll get a compilation error.

To learn more about compilation visit:

https://brainly.com/question/27882492

#SPJ11

Answer Immediaetly Please

Answers

Given SV = 15, UV = 30, and RS = 55, we found TV by using the fact that triangles TRU and SUC are similar. The length of TV is 110.

In the given diagram, we have a triangle TRS with a line UV that is parallel to the base RS. We are given that SV = 15, UV = 30, and RS = 55, and we need to find the length of TV.

To find TV, we can use the fact that UV is parallel to RS, which means that triangles TRU and SUV are similar.

Using the similarity of triangles TRU and SUC, we can set up the following proportion

TV / RS = UV / SV

Substituting the given values

TV / 55 = 30 / 15

Simplifying

TV / 55 = 2

Multiplying both sides by 55

TV = 110

Therefore, the length of TV is 110.

To know more about similarity of triangles:

https://brainly.com/question/29191745

#SPJ1

a recent study at a university showed that the proportion of students who commute more than 15 miles to school is 25%. suppose we have good reason to suspect that the proportion is greater than 25%, and we carry out a hypothesis test. state the null hypothesis h0 and the alternative hypothesis h1 that we would use for this test.H0:H1:

Answers

Answer:

las cañaverales son extenso y hay numerosos

The null hypothesis, H0, is that the proportion of students who commute more than 15 miles to school is equal to or less than 25%. The alternative hypothesis, H1, is that the proportion is greater than 25%.

H0: Proportion of students who commute more than 15 miles to school ≤ 25%
H1: Proportion of students who commute more than 15 miles to school > 25%
In this hypothesis test, we will be using the following terms:

- Null Hypothesis (H0): The proportion of students who commute more than 15 miles to school is equal to 25%.
- Alternative Hypothesis (H1): The proportion of students who commute more than 15 miles to school is greater than 25%.

To restate the hypotheses:

H0: p = 0.25
H1: p > 0.25

Here, p represents the proportion of students who commute more than 15 miles to school.

To learn more about null hypothesis go to :

https://brainly.com/question/27335001#

#SPJ11

Michael has set up an IRA and will deposit $3,000 at the end of each year from age 25 to age 65. Find the
amount of the annuity if the investment is in a stock fund yielding 7% interest, compounded annually.

$300,000.00
$199,635.28
$598,905.30
$226, 351.17

Answers

The amount (future value) of the annuity, if a $3,000 annual deposit is made in a stock fund yielding 7% interest, compounded annually, is C. $598,905.30.

How the future value is determined:

The future value is determined by compounding the periodic deposits and interests.

Compounding describes a process that charges interest on interest.

The future value can be computed using an online finance calculator as follows:

N (# of periods) = 40 years (65 - 25)

I/Y (Interest per year) = 7%

PV (Present Value) = $0

PMT (Periodic Payment) = $3,000

Results:

Future Value (FV) = $598,905.34

The sum of all periodic payments = $120,000.00

Total Interest = $478,905.34

Learn more about the future value at https://brainly.com/question/27979326.

#SPJ1

The 6th term of an arithmetic sequence is 35, and the 41th term
is 315. The common difference is:
5
35
8
7

Answers

The common difference in the arithmetic sequence is 8.

To find the common difference in the arithmetic sequence, we can use the formula:

An = A1 + (n-1)d

Where An is the nth term, A1 is the first term, n is the position of the term, and d is the common difference.

We are given the 6th term (35) and the 41st term (315). We can set up two equations using the formula:

35 = A1 + 5d (1) (6th term)
315 = A1 + 40d (2) (41st term)

Subtract equation (1) from equation (2) to eliminate A1:

315 - 35 = (A1 + 40d) - (A1 + 5d)
280 = 35d

Now, solve for the common difference (d):

d = 280 / 35
d = 8

The common difference in the arithmetic sequence is 8.

Learn more about "arithmetic sequence": https://brainly.com/question/6561461

#SPJ11

How many x-intercepts appear on the graph of this polynomial function?

f (x) = x Superscript 4 Baseline minus 5 x squared

Answers

The value of x - intercepts are,

⇒ x = ±√5, 0, 0

We have to given that;

The function is,

⇒ f (x) = x⁴ - 5x²

Now, We can find the value of x - intercept as;

⇒ f (x) = x⁴ - 5x²

Plug f (x) = 0

⇒ 0 = x⁴ - 5x²

⇒ x² (x² - 5) = 0

⇒ x² = 0

⇒ x = 0, 0

And, x² - 5 = 0

⇒ x² = 5

⇒ x = ±√5

Thus, The value of x - intercepts are,

⇒ x = ±√5, 0, 0

Learn more about the function visit:

https://brainly.com/question/11624077

#SPJ1

Answer:

C

Step-by-step explanation:

edge 2023

spinner has 4 red 3 blues and one yello what is the theoretical probability that it will spin red then blue

Answers

The theoretical probability of the spinner landing on red then blue is 1/9

Determining the theoretical probability of the spinner landing on red then blue

From the question, we have the following parameters that can be used in our computation:

Colors = 3 i.e. yellow, blue and red

Blue = 1

Red = 1

So, we have

Theoretical probability = Red/Colors * Blue/Colors

Substitute the known values in the above equation, so, we have the following representation

Theoretical probability = 1/3 * 1/3

Evaluate

Theoretical probability = 1/9

Hence, theoretical probability is 1/9

Read more about probability at

brainly.com/question/251701

#SPJ1

A window frame is made of four inner squares like shown below.
Pleaseee helpp

Answers

The perimeter of the outer square in red  is: 320 cm

What is the perimeter of the square?

The perimeter of a square is defined by the formuls:

P = 4 * side length

Now, we are told that each of the internal 4 squares have a perimeter of 160 cm.

Thus:

160 = 4 * side length

side length = 160/4

side length = 40 cm

Now, this means that the side length of the outer square in red is:

Side length = 2 * 40

= 80 cm

Thus:

Perimeter of outer square in red = 4 * 80

= 320 cm

Read more about Perimeter of Square at: https://brainly.com/question/25092270

#SPJ1

11. [0.33/1 Points] DETAILS PREVIOUS ANSWERS Math 110 Course Resources - Implicit Differentiaion & Related Rates Course Packet on computing elasticity of demand using implicit differentiation The demand function for SkanDisc 2GB thumb drives is given by P = 5(x + 4) "4 where p is the wholesale unit price in dollars and x is the quantity demanded each week, measured in units of a thousand. Compute the price, p, when x-12. Do not round your answer. 80 Price, p = dollars Use implicit differentiation to compute the rate of change of demand with respect to price,p, when x = 12. Do not round your answer. - 15 Rate of change of demand, x'- thousands of units per dollar I х Compute the elasticity of demand when x - 12. Do not round your answer. 9 Elasticity of Demand x

Answers

The price when x = 12 is 80 dollars.

The elasticity of demand, according to the given conditions,  when x = 12 is 0.0625

To compute the price, p, when x = 12, we plug in x = 12 into the demand function P = 5(x + 4) "4:

P = 5(12 + 4) "4
P = 80

So the price when x = 12 is 80 dollars.

To compute the rate of change of demand with respect to price, p, we use implicit differentiation. Differentiating both sides of the demand function P = 5(x + 4) "4 with respect to p, we get:

dP/dp = 5(dx/dp)

Solving for dx/dp, we get:

dx/dp = (dP/dp) / 5

We know that dP/dx = 5, since that is the coefficient of x in the demand function. So when x = 12, we have:

dP/dx = 5
dP/dp = (dP/dx)(dx/dp) = 5(dx/dp)

Substituting in dP/dp = -15 (since we want the rate of change of demand with respect to price, not quantity), we get:

-15 = 5(dx/dp)
dx/dp = -3

So the rate of change of demand with respect to price, when x = 12, is -3 thousand units per dollar.

To compute the elasticity of demand when x = 12, we use the formula:

Elasticity of Demand = (% change in quantity demanded) / (% change in price)

We can find the % change in quantity demanded by using the derivative of the demand function. We have:

P = 5(x + 4) "4
dP/dx = 5
dP/dx = 5(x + 4)"5(dx/dx) = 5(12 + 4)"5(dx/dx)
dx/dx = (dP/dx) / (5(x + 4)"5) = 1 / (x + 4)"5

So when x = 12, we have:

dx/dx = 1 / (12 + 4)"5 = 1/16

This means that a 1% increase in quantity demanded corresponds to a 1/16% increase in x. Similarly, a 1% decrease in quantity demanded corresponds to a 1/16% decrease in x.

To find the % change in price, we can use the fact that the demand function is:

P = 5(x + 4) "4

This means that a 1% increase in price corresponds to a 1% increase in P, since there are no other variables involved in the equation. Similarly, a 1% decrease in price corresponds to a 1% decrease in P.

So we have:

% change in quantity demanded = 1/16%
% change in price = 1%

Plugging these into the formula for elasticity of demand, we get:

Elasticity of Demand = (% change in quantity demanded) / (% change in price)
Elasticity of Demand = (1/16%) / (1%)
Elasticity of Demand = 1/16

So the elasticity of demand when x = 12 is 1/16 or 0.0625.

To learn more about elasticity of demand visit : https://brainly.com/question/1048608

#SPJ11

The square of a positive number is 42 more than the number itself. What is the number?

Answers

The number we're looking for is 7.

Let's call the number we're looking for "x". According to the problem, the square of the number is 42 more than the number itself. In equation form, this can be written as:

[tex]x^2[/tex] = x + 42

To solve for x, we want to get all the terms on one side of the equation. We can start by subtracting x + 42 from both sides:

[tex]x^2[/tex] - x - 42 = 0

Now we have a quadratic equation. We can solve it by factoring or by using the quadratic formula. Let's use factoring. We want to find two numbers that multiply to -42 and add up to -1 (since the coefficient of x is -1). One possible pair of numbers is -7 and 6, since -7 × 6 = -42 and -7 + 6 = -1. So we can rewrite the equation as:

(x - 7)(x + 6) = 0

This tells us that either x - 7 = 0 or x + 6 = 0. Solving for x in each case, we get:x = 7 or x = -6

We're looking for a positive number, so the solution is x = 7. Therefore, the number we're looking for is 7.

Learn more about quadratic equation,

https://brainly.com/question/30098550

#SPJ4

A 12-foot pole is supporting a tent and has a rope attached to the top

Answers

The expression that represent the length of the rope is 10 / cos 40° =  13.1 feet

How to find the expression that show the length of the rope?

A 12-foot pole is supporting a tent and has a rope attached to the top. The rope is pulled straight and the other end is attached to a peg two foot above the ground.

This situation forms a right angle triangle. Therefore, let's find the expression that shows the length of the rope using trigonometric ratios.

Hence,

cos 40 = adjacent / hypotenuse

adjacent side = 10 ft

Therefore,

cos 40° = 10 / x

where

x = length of the rope

cross multiply

x = 10 / cos 40°

x = 10 / 0.76604444311

x = 13.0548302872

x = 13.1 feet

learn more on right triangle here: https://brainly.com/question/28281614

#SPJ1

PLEASE ANSWER!!!! 20 POINTS
--
Find the mean x of the data 16, 31, 38, 24, 36

Answers

Answer:

Find the mean x of the data 16, 31, 38, 24, 36

16 + 31 + 38 + 24 + 36

= 145

145 ÷ 5

= 29

Step-by-step explanation:

You're welcome.

Which expression is equivalent to x^{2}-36

Answers

The answer is

(-x-6i)(x-6i)

At a craft shop, a painter decided to paint a welcome sign to take home. An image of the sign is shown.

A five-sided figure with a flat top labeled 5 and one-half feet. A height labeled 4 feet. The length of the entire image is 9 ft. There is a point coming out of the right side of the image that is created by two line segments.

What is the area of the sign?

19 square feet
22 square feet
29 square feet
36 square feet

Answers

The area of the composite figure is 29 feet squared.

How to find the area of a composite figure?

A five-sided figure with a flat top labelled 5 and one-half feet.  A height labelled 4 feet. The length of the entire image is 9 ft.

Therefore, the area of the composite figure can  be found as follows;

The figure can be divide into two shapes which are rectangle and a triangle.

Hence,

area of the composite figure = area of the rectangle + area of the triangle

area of the rectangle = 4 × 5.5 = 22 ft²

area of the triangle = 1 / 2 bh

where

b = base h = height

area of the triangle = 1 / 2 × 4 × (9 - 5.5)

area of the triangle = 1 / 2 × 4 × 3.5

area of the triangle = 14 / 2

area of the triangle = 7 ft²

Therefore,

area of the composite figure = 22 + 7

area of the composite figure = 29 ft²

learn more on area here: https://brainly.com/question/31272718

#SPJ1

38. A new apartment complex with 90 one-bedroom apartment units and 100 two-bedroom apartment units was built near a lake. Rental prices that will provide full occupancy are estimated at $1200 for one-bedroom units and $1800 for two-bedroom units. A market survey suggests that for every $20 increase in the price of a one-bedroom unit one less customer will sign a lease and for every $60 increase in the price of a two-bedroom unit two less customers will sign a lease. What rental price should the manager charge to maximize revenue?

Answers

The required manager should charge $1600 for one-bedroom units and $2250 for two-bedroom units to maximize revenue.

Let x be the number of $20 increases in the price of a one-bedroom unit, and y be the number of $60 increases in the price of a two-bedroom unit. Then the rental prices for one-bedroom and two-bedroom units can be expressed as:

One-bedroom price = $1200 + $20x

Two-bedroom price = $1800 + $60y

The total number of customers for one-bedroom units is 90 minus the number of customers lost due to the price increase, which is x. Similarly, the total number of customers for two-bedroom units is 100 minus the number of customers lost due to the price increase, which is 2y. Therefore, the total revenue can be expressed as:

Revenue = (90 - x) * ($1200 + $20x) + (100 - 2y) * ($1800 + $60y)

Expanding and simplifying this expression, we get:

Revenue = 216000 + 9600x - 240x² + 180000 + 108000y - 7200y²

Collecting like terms, we get:

Revenue = -240x² - 7200y² + 9600x + 108000y + 396000

To find the rental price that maximizes revenue, we need to find the values of x and y that maximize the revenue. We can do this by taking partial derivatives of the revenue function with respect to x and y and setting them equal to zero:

dRevenue/dx = -480x + 9600 = 0

dRevenue/dy = -14400y + 108000 = 0

Solving for x and y, we get:

x = 20

y = 7.5

Therefore, the rental prices that maximize revenue are:

One-bedroom price = $1200 + $20x = $1600

Two-bedroom price = $1800 + $60y = $2250

So the manager should charge $1600 for one-bedroom units and $2250 for two-bedroom units to maximize revenue.

Learn more about derivatives here:

https://brainly.com/question/30365299

#SPJ1

The plane passing through the point P(1,3,4) with normal vector 2i+63 +7k has equation 5+3y+42=48 · Answer Ο Α True O B False

Answers

The statement  "The plane passing through the point P(1,3,4) with normal vector 2i+63 +7k has equation 5+3y+42=48" is false because the equation '5+3y+42=48' given in the question is wrong.

The equation of a plane passing through a point P(x1, y1, z1) with a normal vector N(A, B, C) is given by: A(x-x1) + B(y-y1) + C(z-z1) = 0In this case, the point P is (1, 3, 4) and the normal vector is 2i+63 +7k. So, the equation of the plane passing through P is:
2(x-1) + 63(y-3) + 7(z-4) = 0 Simplifying this equation, we get:
2x + 63y + 7z = 200This is not the same as the equation given in the question, which is:
5+3y+42=48So, the answer is False.

know more about normal vector here: https://brainly.com/question/31493378

#SPJ11

What is the common ratio?

n f(n)

1 300

2 375

3 468.75

4 585.9375

Write an explicit rule for the geometric sequence

What is f(12)?

Answers

The common ratio is 1.25. An explicit rule for the geometric sequence is  f(n) = 300(1.25)ⁿ⁻¹ . The value of f(12) is 5,722.05.

To find the common ratio of the sequence, we need to divide each term by the previous term. For example, to find the common ratio between the first two terms:

375/300 = 1.25

Similarly, we can find the common ratio between the second and third terms:

468.75/375 = 1.25

And the common ratio between the third and fourth terms:

585.9375/468.75 = 1.25

Since the common ratio is the same for each pair of adjacent terms, we can conclude that the explicit rule for the geometric sequence is:

f(n) = 300(1.25)ⁿ⁻¹

To find f(12), we can simply substitute 12 for n in the formula:

f(12) = 300(1.25)¹²⁻¹

f(12) = 300(1.25)¹¹

f(12) = 300(19.0735)

f(12) = 5,722.05

Therefore, f(12) is 5,722.05.

To learn more about sequence click on,

https://brainly.com/question/29125671

#SPJ1

A paper bag has seven colored marbles. The marbles are pink, red, green, blue, purple, yellow, and orange. List the sample space when choosing one marble.

S = {1, 2, 3, 4, 5, 6}
S = {purple, pink, red, blue, green, orange, yellow}
S = {g, r, b, y, o, p}
S = {green, blue, yellow, orange, purple, red}

Answers

the answer to your math question is S = {green, blue, yellow, orange, purple, red}

Find the slope for the line that passes through the points (-2,5) and (1,0)

Answers

Answer:

[tex]m=\frac{-5}{3}[/tex]

Step-by-step explanation:

Pre-Solving

We want to find the slope between the points (-2,5) and (1,0).

The slope (m) can be found using the formula [tex]\frac{y_2-y_1}{x_2-x_1}[/tex], where [tex](x_1,y_1)[/tex] and [tex](x_2,y_2)[/tex] are points.

Solving

We are already given the values of the points, but let's label their values to avoid any confusion and mistakes.

[tex]x_1=-2\\y_1=5\\x_2=1\\y_2=0[/tex]

Now, substitute into the formula.

[tex]m=\frac{y_2-y_1}{x_2-x_1}[/tex]

[tex]m=\frac{0-5}{1--2}[/tex]

Simplify this to:

[tex]m=\frac{0-5}{1+2}[/tex]

[tex]m=\frac{-5}{3}[/tex]

The slope is -5/3.

6. Find the absolute minimum and absolute maximum values of f(x) = 3x^4 - 4x^3-36^x2, -3 ≤x≤5.

Answers

The absolute minimum and absolute maximum values of the function f(x) = 3x^4 - 4x^3 - 36x^2 on the interval [-3, 5] are -283 and 81, respectively. To get the absolute minimum and absolute maximum values of the function f(x) = 3x^4 - 4x^3 - 36x^2 on the interval [-3, 5].


Step 1: Find the critical points by taking the derivative of the function and setting it equal to zero.
f'(x) = 12x^3 - 12x^2 - 72x
Step 2: Factor the derivative.
f'(x) = 12x(x^2 - x - 6)
Step 3: Solve for x to find the critical points.
x = 0, x = -1, x = 6
Step 4: Evaluate the function at the critical points and endpoints of the interval.
f(-3) = 81
f(0) = 0
f(-1) = 43
f(5) = -283
Step 5: Identify the absolute minimum and absolute maximum values.
The absolute minimum value of f(x) is -283 at x = 5.
The absolute maximum value of f(x) is 81 at x = -3.
So, the absolute minimum and absolute maximum values of the function f(x) = 3x^4 - 4x^3 - 36x^2 on the interval [-3, 5] are -283 and 81, respectively.

Learn more about absolute maximum value here, https://brainly.com/question/19921479

#SPJ11

The tree diagram represents an
experiment consisting of two trials.
S
A
B
.4 C
6
13
D
C
D

Answers

The required probability is P(A and C) is 0.2 which is represented in the tree diagram.

What is probability?

Probability is defined as the possibility of an event being equal to the ratio of the number of favorable outcomes and the total number of outcomes.

The given tree diagram represents an experiment consisting of two trials.

The tree diagram represents an experiment consisting of two trials. In this case, the probability of event A and event C occurring is represented by the intersection of branches A and C in the tree diagram.

This probability can be calculated by multiplying the probability of each individual event together.

As per the given question, we have

P(A) = 0.5

P(C|A) = 0.4

So, P(A and C) = 0.5 × 0.4 = 0.2

Thus, the required probability is P(A and C) is 0.2

Learn more about probability on https://brainly.com/question/30034780

#SPJ1

You pay $1 to play a game in which you roll one fair die. If you roll a 6 on the first roll, you win $5. If you roll a 1 or a 2, you win $2. If not, you lose money.

a. Start with $10. Play the game 10 times. Keep track of the number of times you win and determine the amount of money you have left, at the end of the game.

b. Create a probability distribution for this game.

c. Find the expected value for this game.

Answers

After 10 rolls, we won 3 times and lost 7 times, and we have $11 left.

The probability distribution for this game is:

Outcome Probability

Lose 2/3

Win $2 1/6

Win $5 1/6

How to explain the probability

It should be noted that to calculate the anticipated value, multiply the likelihood of each scenario by its payment and add them together:

E(X) = (2/3) * (-1) + (1/6) * 2 + (1/6) * 5 = -2/3 + 1/3 + 5/6 = 1/2

As a result, the expected value of this game is $0.50. This indicates that if you play it frequently, you can expect to win $0.50 each game on average. However, you could win or lose money in any particular game.

Learn more about probability on

https://brainly.com/question/24756209

#SPJ1

In the diagram shown, points A and B have been dilated from center O . |AB|=12 and |A′B′|=8 . A ray starts at point O and passes through points A prime and A. A second ray starts at O and passes through points B prime and B. Segments A prime B prime and A B are drawn between the rays. What is the scale factor r so that dilation from center O maps segment AB to segment A′B′ ?

Answers

Answer:

Step-by-step explanation:

i dont know how to do this help me  im on a test and cant do this

Other Questions
Which statement is false?a. The source and application of funds technique is a variation of the net worth method that shows increases and decreases in a taxpayer's accounts at the end of the year.b. Often the IRS agent will use the expenditure approach when a taxpayer is spending income lavishly rather than purchasing assets or investments.c. The expenditure approach is similar to the Cash T, except that the data used is the increases and decreases in the taxpayer's accounts.d. The source and application of funds method is a variation of the bank deposit method.e. None of the above For what value of A, the binary number 1000A12 represents 35? 1:WHAT ROLE DID SOUTH AFRICAN WOMEN PLAY AGAINST THE VIOLATION OF HUMAN RIGHTS FROM THE 1950s TO 1960s? Which of the following leads to strong coordination across functional areas and greater flexibility in responding to changes in the environments?a. Reengineeringb. Layoffsc. Downsizingd. A temporary committeee. An ad hoc committee When hired at a new job selling jewelry, you are given two pay options: Option A: Base salary of $19,000 year with a commission of 12% of your sales Option B: Base salary of $28,000 a year with a commission of 8% of your sales How much jewelry would you need to sell for option A to produce a larger income? Soil in good condition for plant growth will be composed of ____. A) ~95 percent organic matter B) less than 1 percent water C) ~50 percent water D) less than 1 percent air E) ~45 percent mineral matter you are given two solutions. solution x has a volume of 50.0 ml and contains 0.0060 moles of imidazole and 0.0040 moles of imidazolium chloride. solution y has a volume of 50.0 ml and contains 0.060 moles of imidazole and 0.040 moles of imidazolium chloride. what are the ph values of solutions x and y? how does nursing care for a patient in a crisis differ from nursing in other situations? A 54 year old man refuses your care and treatment despite his wife's pleas. He refuses to sign a refusal of care form. Now he is telling you to leave his house immediately.A) Contact medical direction for permission to cease care without a signed refusal of care form.B) Leave the scene after asking his wife to sign the refusal form as a witness.C) Call police to the scene for possible intervention.D) Leave the scene, documenting in your reports his refusal of care. CN X, contols muscles in the pharynx and larynx, influences function of hte abdominal and thoracic visera, and carries sensory information from the abdomen and thorax to the brain. It's fxn can be assessed by inducing gag reflex.true or fasle Which of these describes the new view?A.) earth was the center of the universe B.) the church was the accepted authority C.) knowledge was based on tradition D.) travel to new lands changed minds Q2: Find a root, using Bisection method and false position methods of an equation f(x)=2x-2x-5 between 1 and 3 (5 iteration) f(x)=2*cos(x)-x between 1 and 3 (5 iteration) Hint(Solution correct upto 5 digit) kinston industries has come up with a new mountain bike prototype and is ready to go ahead with pilot production and test marketing. the pilot production and test marketing phase will last for one year and cost $500,000. your management team believes that there is a 50% chance that the test marketing will be successful and that there will be sufficient demand for the new mountain bike. if the test-marketing phase is successful, then kinston industries will invest $3 million in year one to build a plant that will generate expected annual after tax cash flows of $400,000 in perpetuity beginning in year two. if the test marketing is not successful, kinston can still go ahead and build the new plant, but the expected annual after tax cash flows would be only $200,000 in perpetuity beginning in year two. kinston has the option to stop the project at any time and sell the prototype mountain bike to an overseas competitor for $300,000. the appropriate discount rate for the project is 10%. assuming that kinston has the ability to sell the prototype in year one for $300,000, the npv of the kinston industries mountain bike project is closest to: group of answer choices 75,000 60,000 45,000 90,000 In two or three sentences, explain the evolution of Pittsburgh as related to the function of the city. Lipid A is a(n) (cytotoxin/endotoxin/exotoxin) that stimulates the body to release chemicals that cause fever, inflammation, diarrhea, hemorrhaging, shock, and blood coagulation. which of the following is not a factor commonly affecting channel choice and supply chain management? a. producer factors. b. customer factors. c. product factors. Which 2 religions were the fastest growing during the Second Great Awakening? How many moles of aluminum will be used when reacted with 1.35 moles of oxygen based on this chemical reaction? __Al + ___ O2 2Al2O3 true or false?buspirone can be used long-term for anxiety What diagnostic workup of a patient with chest pain and stomach pain?